The Student Room Group

proof of the differential of sin x

I was trying to get tot he derivative of sin x from first principles. I realise that most proofs of this use a different expansion for sin(a+b), but it should work with this one too presumably?

Could do with some help...

let
y=sinxy=\sin x

let h be a small increment to x

dydx=sin(x+h)sinxh\dfrac{dy}{dx}=\dfrac{\sin(x+h)-\sin x}{h}

dydx=(sinxcosh+sinhcosx)sinxh\dfrac{dy}{dx}=\dfrac{(\sin x\cos h + \sin h \cos x)-\sin x}{h}

as h tends to zero

limh0  sinxcosh=sinx\displaystyle\lim_{h\rightarrow 0}\;\sin x\cos h=\sin x

limh0  sinhcosx=0\displaystyle\lim_{h\rightarrow 0}\;\sin h\cos x=0

so now I have 0/0 ?

:confused:
(edited 13 years ago)

Scroll to see replies

Reply 1
Yeah, that almost always happens when you're trying to calculate derivatives by limits. You'll need to prove that sinhh1\frac{\sin h}{h} \to 1.
Original post by Zhen Lin
Yeah, that almost always happens when you're trying to calculate derivatives by limits. You'll need to prove that sinhh1\frac{\sin h}{h} \to 1.


You use the squeeze theorem, right?
Reply 3
And it is usually shown by a geometric argument that (sinh/h) tends to 1 - Google it, it's well documented.
Reply 4
Original post by boromir9111
You use the squeeze theorem, right?


If that's the same as the Sandwich theorem (i.e. if a < b < c and a and c tend to 0, then b tends to 0), then yeah.
Reply 5
You also need to prove that limh0cosh1h=0\displaystyle \lim_{h \to 0}\frac{\cos h - 1}{h } = 0. Hint: Multiplication by a conjugate.




Edit:
Original post by Plato's Trousers
x


If you're that interested in alternate proofs, a completely different proof can be found on page 21 :p:
(edited 13 years ago)
Original post by Zhen Lin
Yeah, that almost always happens when you're trying to calculate derivatives by limits. You'll need to prove that sinhh1\frac{\sin h}{h} \to 1.


yes, I have seen the "official" proofs. But I guess my question was why it doesn't work this way?

(don't all roads lead to Rome?)
Original post by Swayum
If that's the same as the Sandwich theorem (i.e. if a < b < c and a and c tend to 0, then b tends to 0), then yeah.


:yes:
Reply 8
Original post by Plato's Trousers
yes, I have seen the "official" proofs. But I guess my question was why it doesn't work this way?

(don't all roads lead to Rome?)


It does work. But when you encounter 0/0, you hit a road block. 0/0 could be anything! You can't then just plug in h = 0 into the numerator and denominator and hope that life just works out. You have to use a different argument to show what the limit is. What we're suggesting is that you split your expression:

(sinxcosh+sinhcosx)sinxh=sinx(cosh1)h+sinhcosxh\dfrac{(\sin x\cos h + \sin h \cos x)-\sin x}{h} = \dfrac{\sin x(\cos h - 1)}{h} + \dfrac{\sin h \cos x}{h}

And take limits separately on those two terms.
(edited 13 years ago)
Reply 9
Also, it is bad to write

dydx=sin(x+h)sinxh\dfrac{dy}{dx}=\dfrac{\sin(x+h)-\sin x}{h}

This isn't true.

dydx=limh0  sin(x+h)sinxh\dfrac{dy}{dx}=\lim_{h \rightarrow 0}\;\dfrac{\sin(x+h)-\sin x}{h} is true

If you don't want to write lim all the time, then you say δyδx=sin(x+h)sinxh\dfrac{\delta y}{\delta x}=\dfrac{\sin(x+h)-\sin x}{h}

And take limits on both sides at the end saying that delta y/delta x tends to dy/dx.
(edited 13 years ago)
Original post by Swayum
It does work. But when you encounter 0/0, you hit a road block. 0/0 could be anything! You can't then just plug in h = 0 into the numerator and denominator and hope that life just works out. You have to use a different argument to show what the limit is. What we're suggesting is that you split your expression:

(sinxcosh+sinhcosx)sinxh=sinx(cosh1)h+sinhcosxh\dfrac{(\sin x\cos h + \sin h \cos x)-\sin x}{h} = \dfrac{\sin x(\cos h - 1)}{h} + \dfrac{\sin h \cos x}{h}

And take limits separately on those two terms.


aha! I see.

So

limh0sinx(cosh1)h=0\displaystyle\lim_{h\rightarrow 0} \dfrac{\sin x(\cos h - 1)}{h} = 0

and

limh0sinhcosxh=cosx\displaystyle\lim_{h\rightarrow 0} \dfrac{\sin h \cos x}{h}=\cos x

since

limh0sinhh=1\displaystyle\lim_{h\rightarrow 0} \dfrac{\sin h}{h}=1

Thanks
Original post by Swayum
Also, it is bad to write

dydx=sin(x+h)sinxh\dfrac{dy}{dx}=\dfrac{\sin(x+h)-\sin x}{h}

This isn't true.

dydx=limh0  sin(x+h)sinxh\dfrac{dy}{dx}=\lim_{h \rightarrow 0}\;\dfrac{\sin(x+h)-\sin x}{h} is true



Sorry. Yes, I knew that. I was being lazy.
Reply 12
Original post by Plato's Trousers
latex]\displaystyle\lim_{h\rightarrow 0} \dfrac{\sin x(\cos h - 1)}{h} = 0



Yes, but why?
Original post by SimonM
Yes, but why?


because as h -> 0, cos h -> 1, so the bracket disappears.
Reply 14
Original post by Plato's Trousers
because as h -> 0, cos h -> 1, so the bracket disappears.


Yeah, see that logic is wrong. I think you need to read the thread again to try to get to the bottom of how limits work.
Reply 15
Original post by Plato's Trousers
because as h -> 0, cos h -> 1, so the bracket disappears.


But the denominator is going to 0 as well, so you're stuck in a 0/0 situation again! Your argument would be fine if we were dealing with (1 - cosh)/(1 + h): over here you can let h = 0 in the numerator and denominator and conclude the limit is 0, but not when you get 0/0 (or infinity/infinty). That's what Simon's getting at.

(sorry about being pedantic before, btw, just wanted to make sure you knew)
(edited 13 years ago)
Original post by SimonM
Yeah, see that logic is wrong. I think you need to read the thread again to try to get to the bottom of how limits work.


oh. :frown:


Original post by Saichu
You also need to prove that limh0cosh1h=0\displaystyle \lim_{h \to 0}\frac{\cos h - 1}{h } = 0. Hint: Multiplication by a conjugate.


..ahh, you mean this? Ok, I'll try to prove that then.


Original post by Swayum

(sorry about being pedantic before, btw, just wanted to make sure you knew)


no probs. Pedantic is good. I like pedantic (otherwise I wouldn't be doing maths :smile: )
ok, so here's my attempt. Is this correct?

given

cosh1h\dfrac{\cos h -1}{h}

multiply top and bottom by cosh+1\cos h +1


cos2h1h(cosh+1)=sin2hh(cosh+1)\dfrac{\cos^2 h -1}{h(\cos h +1)}=\dfrac{-\sin^2 h}{h(\cos h +1)}

divide top and bottom by h

sin2hhcosh+1\dfrac{\frac{-\sin^2 h}{h}}{\cos h+1}

but

limh0sin2hh=sinh\displaystyle\lim_{h\rightarrow 0}\dfrac{-\sin^2 h}{h}=-\sin h

and

limh0cosh+1=2\displaystyle\lim_{h\rightarrow 0} \cos h+1=2

so

limh0cosh1h=sinh2\displaystyle\lim_{h\rightarrow 0} \dfrac{\cos h -1}{h}=-\dfrac{\sin h}{2}

and since

limh0sinh2=0\displaystyle\lim_{h\rightarrow 0} -\dfrac{\sin h}{2}=0

then

limh0cosh1h=0\displaystyle\lim_{h\rightarrow 0} \dfrac{\cos h -1}{h}=0
(edited 13 years ago)
Reply 18
Original post by Plato's Trousers

limh0sin2hh=sinh\displaystyle\lim_{h\rightarrow 0}\dfrac{\sin^2 h}{h}=-\sin h


This is nonsense. h is a dummy variable, it can't still appear in the RHS. If you can fix this, then yes, you're heading in the right direction
Original post by SimonM
This is nonsense. h is a dummy variable, it can't still appear in the RHS. If you can fix this, then yes, you're heading in the right direction


actually, yes, I was just re-reading it and wasn't happy with that part. :frown:

Quick Reply

Latest